Instructions

What approximate value will come in place of the question mark(?) in the given questions ? (You are not expected to calculate the exact value

Question 34

815.002 + 29.98 —53.998 + 3.01^{2} = ?

Solution

Round of the values to the nearest whole number.
815 + 30 - 54 + 3$$^2$$ = 815 + 30 - 54 + 9 = 800
Option D is the correct answer.


Create a FREE account and get:

  • Banking Quant Shortcuts PDF
  • Free Banking Study Material - (15000 Questions)
  • 135+ Banking previous papers with solutions PDF
  • 100+ Online Tests for Free

cracku

Boost your Prep!

Download App